Let A B C be a triangle in which A B = A C . Suppose the orthocentre of the triangle lies on the incircle.
Find B C A B .
This section requires Javascript.
You are seeing this because something didn't load right. We suggest you, (a) try
refreshing the page, (b) enabling javascript if it is disabled on your browser and,
finally, (c)
loading the
non-javascript version of this page
. We're sorry about the hassle.
Let the orthocenter, incenter and the mid-point of B C be H , I and M respectively; the radius of the incircle be r , B M = C M = a and ∠ M A B = ∠ M A C = θ .
We note that ∠ H B M = ∠ H C M = θ . Then:
tan θ = B M H M = a 2 r
Also that:
sin θ 2 cos 2 θ − sin 2 θ 2 − 3 sin 2 θ ⇒ 3 sin 2 θ + sin θ − 2 ( 3 sin θ − 2 ) ( sin θ + 1 ) ⇒ sin θ = A M − I M r = tan θ a − r r = r tan θ a − 1 1 = tan 2 θ 2 − 1 1 = 2 cos 2 θ − sin 2 θ sin 2 θ = sin θ = sin θ = 0 = 0 = { 3 2 − 1 θ < 9 0 ∘ rejected
Therefore B C A B = A B B C 1 = A B 2 B M 1 = 2 sin θ 1 = 4 3 = 0 . 7 5
I got sin(alpha/2) properly but I divided wrongly
I'm going to give two solutions. The first is done by using the Olympiad knowledge; the second is done by using coordinate geometry, from which even a junior high-school student should understand very well.
Solution:
Let D and r be the midpoint of B C and the inradius of △ A B C respectively. Since A B = A C , the incentre I , circumcentre O and orthocentre H all lie on the median A D . Then D I = r . For H lying on the incircle of △ A B C , I H = r and hence D H = D I + I H = 2 r .
Let A D produced, B H produced and C H produced meet ⊙ ( A B C ) , A C and A B at H ′ , E and F respectively.
By the definition of the orthocentre, ∠ C F A = ∠ B E A = 9 0 ∘ . Since O lies on A H ′ , A H ′ is the diameter of ⊙ ( A B C ) and thereby ∠ A B H ′ = ∠ A C H ′ = 9 0 ∘ . Then B H ′ / / C H and H ′ C / / B H . It follows that B H C H ′ is a parallelogram. The diagonals B C and H H ′ meet at D and therefore H ′ D = D H = 2 r .
It is well-known in the Olympiad community that H ′ is the centre of ⊙ ( B I C ) . Consequently, B H ′ = I H ′ = I D + D H ′ = 3 r . It is readily seen that △ A B D ∼ △ B H ′ D . So the proportionality gives
⟹ ⟹ B H ′ A B B D A B B C A B = H ′ D B D = H ′ D B H ′ = 2 r 3 r = 2 3 = 2 B D A B = 4 3 = 0 . 7 5 , as required. []
.
.
Alternative Solution:
Introduce a coordinate system such that A = ( b , h ) , B = ( 0 , 0 ) , C = ( 2 b , 0 ) .
Let M = ( b , 0 ) . It is clear that A M is a median. Since △ A B C is an isosceles triangle, both incentre I and orthocentre H lie on A M .
Let r be the inradius of △ A B C . Then I = ( b , r ) . Since H lies on A M and the incircle of △ A B C , H = ( b , 2 r ) .
Note that C H ⊥ A B , by definition of the orthocentre. This means that
m C H ⋅ m A B 2 b − b 0 − 2 r ⋅ b − 0 h − 0 b 2 = − 1 = − 1 = 2 h r ( 1 )
Consider the semi-perimeter of △ A B C : s △ A B C = 2 2 b + b 2 + h 2 = b 2 + b 2 + h 2 , and the area of △ A B C : ( A B C ) = b h . As ( A B C ) = r s △ A B C , we have
r = b + b 2 + h 2 b h ( 2 )
Plugging ( 2 ) into ( 1 ) , we obtain
b 2 2 b h 2 2 h 2 ( 2 h 2 − b 2 ) 2 4 h 4 − 5 b 2 h 2 h 2 ( 4 h 2 − 5 b 2 ) h 2 = 2 h ( b + b 2 + h 2 b h ) = b 3 + b 2 b 2 + h 2 = b 2 + b b 2 + h 2 = b 4 + b 2 h 2 = 0 = 0 = 4 5 b 2 since b = 0 since h = 0
Hence A B = b 2 + h 2 = b 2 + 4 5 b 2 = 2 3 b
and thus the required ratio: B C A B = 2 b 2 3 b = 4 3 = 0 . 7 5 . []
Let the foot of the perpendicular from A to B C be M .
Let H , I be the orthocentre and incentre respectively.
(I)Since in an isosceles triangle orthocentre and incentre are collinear .
Proof of statement I :
△ ( A B M ) is congruent to △ ( A C M ) (By R H S test ) A B = A C , A M = A M , ∠ ( A M B ) = ∠ ( A M C )
Therefore M is a midpoint of B C . Let J be the intersection of incircle with B C .
Therefore ∠ ( I J C ) = 9 0 ° . 2 B J = A B + B C − A C , 2 C J = B C + A C − A B
But, A B = A C
Therefore B J = C J , B J + C J = B C .
This implies J is the midpoint of B C , but we proved that M is the midpoint of B C .
Therefore J , M must coincide.
This proves that I lies on A M .
Therefore A , M , H , I are colinear.
Since A B = A C , ∠ ( B ) = ∠ ( C ) Now t a n ( 2 B ) = B M I M . . . . . . . ( 1 ) ) ∠ ( H B C = ( 9 0 − C ) = ( 9 0 − B )
Therefore t a n ( 9 0 − B ) = B M H M
But 2 I M = H M since H , I , M are collinear and also lie on a circle with I as the centre .
hence C o t ( B ) = B M 2 I M . . . . . . . ( 2 )
Dividing ( 1 ) by ( 2 ) We get 2 1 = t a n ( B ) t a n ( 2 B )
After solving this we get, t a n ( B ) = 2 5
But ( 9 0 − 2 A ) = B Therefore t a n ( 2 A ) = 5 2
This implies s i n ( 2 A ) = 3 2 . . . . . . ( 3 )
In triangle A B M s i n ( A / 2 ) = B M / A B = B C / 2 A B . . . . ( 4 )
Therefore using ( 3 ) , ( 4 ) we get
B C A B = 4 3
Let AD be the perpendicular angle bisector.we know HD=2RcosBcosC and A/Q HD=2r. Equating we get the desired result.
My solution uses the coordinates system. I used GeoGebra to present this solution. You can see it at the following link
This problem is from the INMO 2016.
I suggest you to draw a diagram first.
Let angle ABC = x. let D be the foot of perpendicular from A to BC. Let I be the incentre and H be the orthocentre.
Angle IBM = x/2 , angle HBD = 90 - x.
HD = 2ID.
tan (x/2) = ID/BD. tan (90-x) = HD/BD.
This gives the value of tan (x/2), and hence the values of all other angles.
Problem Loading...
Note Loading...
Set Loading...
Since the triangle is isosceles, A , I , H are collinear, where I , H are the incenter and orthocenter respectively. Let A H ∩ B C , ⊙ ( A B C ) = D , E . It is well known that D E = D H , thus H lies on the the incircle implies D E = D H = 2 I D . Using an incenter property from triangles - incenter , we know
I D A I = D E E I = D E I D + D E = 2 3 .
Hence by the angle bisector theorem, B C A B = 2 B D A B = 2 I D A I = 4 3